Evaluate - What does the test want?

This topic has expert replies
Newbie | Next Rank: 10 Posts
Posts: 9
Joined: Wed Dec 29, 2010 2:58 pm

Evaluate - What does the test want?

by Derek1988 » Fri Dec 31, 2010 5:21 am
Premise: I am not a native english speaker. I have the TOEFL, a "regular" B2. I do know what evaluate means, but I miss 1 every 2 questions with "evaluate". I just don't grasp What I have got to search inside the initial statement.
I do know how to approach the strenghten, weaken etc questions, but I don't have an approach for this kind of question - probably because I don't know what evaluate means in the question.

Can anybody help me plz?

Derek

User avatar
GMAT Instructor
Posts: 2193
Joined: Mon Feb 22, 2010 6:30 pm
Location: Vermont and Boston, MA
Thanked: 1186 times
Followed by:512 members
GMAT Score:770

by David@VeritasPrep » Fri Dec 31, 2010 2:56 pm
Can you post an example of the type of question stem you are talking about? In critical reasoning "evaluate" usually means for you to assess the strength of an argument. There are not very many GMAT questions that ask you to assess the strength of the argument, but if you can give an example we can help!
Veritas Prep | GMAT Instructor

Veritas Prep Reviews
Save $100 off any live Veritas Prep GMAT Course

Newbie | Next Rank: 10 Posts
Posts: 9
Joined: Wed Dec 29, 2010 2:58 pm

by Derek1988 » Tue Jan 04, 2011 2:56 am
Of course!

It is illegal to advertise prescription medications in Hedland except directly to physicians, either by mail or in medical journals. A proposed law would allow general advertising of prescription medications. Opponents object that the general population lacks the specialized knowledge to evaluate such advertisements and might ask their physicians for inappropriate medications. But since physicians have the final say as to whether to prescribe a medication for a patient, inappropriate prescriptions would not become more common.

Which of the following would it be most useful to establish in order to evaluate the argument?

(A) Whether advertising for prescription medications might alert patients to the existence of effective treatments for minor ailments that they had previously thought to be untreatable
(B) Whether some people might go to a physician for no reason other than to ask for a particular medication they have seen advertised
(C) Whether the proposed law requires prescription-medication advertisements directed to the general public to provide the same information as do advertisements directed to physicians
(D) Whether advertisements for prescription medications are currently an important source of information about newly available medications for physicians
(E) Whether physicians would give in to a patient's demand for a prescription medication chosen by the patient when the one originally prescribed by the physician fails to perform as desired

OA is E (Kaplan)

It has been alredy posted, but no expert replied and I can't understante why E, aht should I be looking for into the statemant etc.

User avatar
GMAT Instructor
Posts: 2193
Joined: Mon Feb 22, 2010 6:30 pm
Location: Vermont and Boston, MA
Thanked: 1186 times
Followed by:512 members
GMAT Score:770

by David@VeritasPrep » Tue Jan 04, 2011 8:09 am
Very Good! When you first mentioned evaluate I was thinking of something entirely different. This is a Kaplan question, but the last time I took the actual GMAT I got 3 of these questions, so this is an important type.

The key wording here is "most useful to know." This is a subtype of question that we discuss in the Veritas Critical Reasoning 2 book. We group it under weaken because it does rely on the fact that we are missing information and we are to point out what is missing. In this type of question "evaluate" means "to determine how strong it is" but the way that I usually think of this question is, "what fact or question, if known, would either strengthen or weaken this argument?"

In order to attack this question you proceed in the "normal" manner for a question that has a conclusion in the stimulus. This applies to strengthen, weaken, assumption, method of reasoning, and most useful to know questions. The technique here is what I call the "Conclusion -driven technique." On the GMAT when you strengthen or weaken something it is the main conclusion of the argument so you should start and finish there.

Once you have the conclusion you can then ask yourself, "what is the evidence here?" This will point you in the direction of usually just one premise - what I call the "Most Important Premise" or the MIP. The other information will generally be background information.

Once you have the conclusion and the MIP that the conclusion relies upon you can look for the gap between the two. What would cause this evidence (the MIP) to not really lead to the conclusion? This is a technique for a weaken question and it works well here. Once you have determined the information that you are lacking you can look for that in the answer choices.

Let's try these steps for the problem that you have attached:

1) Identify the main conclusion:

The main conclusion for this argument is signaled by the word "would" (which I call a "command conclusion" word along with other commands like, "should, needs, must, and can.")

Conclusion: "inappropriate prescriptions would not become more common."

2) Identify the most important premise (MIP). In this case you have a very helpful tool to distinguish between the MIP and the background information. The signal word "but" indicates that we are going to contradict what has come before. In an argument such as this the MIP and the conclusion must come AFTER the word "but." The information before this word is included in order to then contradict it. Opponents say that people would ask for inappropriate medications "BUT" and then the true reasoning of the argument comes forth to contradict this.

In this case the MIP is therefore "since physicians have the final say as to whether to prescribe a medication for a patient..." So this is the evidence that is supposed to lead to the main conclusion of "inappropriate prescriptions would not become more common."

3) What could go wrong with this evidence leading to this conclusion? Where is the gap in the logic? Well, is it possible that even if a physician has the final say in prescribing that the inappropriate prescriptions might still be more common if the patient sees the advertising?

I have not yet looked at the answer choices, but If we wanted to weaken this argument we would say, "Even though the physician has the final say, the physician will often do what the patient asks and prescribe the inappropriate medicine to keep the patient happy."

If we wanted to strengthen the argument's conclusion we might say, "Physicians always do what they think is best and it does not matter if the patient asks for a medication they have seen advertised since this will not change what the physician does.

4) Let us predict what we are looking for from the correct answer choice. We want an answer choice that indicates the information that we need to know to either more strongly link this MIP to this conclusion, or if we end up with different information, would weaken the link between MIP and conclusion.

So what we are looking for in an answer choice is information that tells us whether the fact that the physician has the final say in which prescriptions actually get written is a protection against inappropriate prescriptions. Specifically I am anticipating something about the physician either listening to a patient that comes in wanting a particular prescription or ignores that patient's request when inappropriate.

Now to the answer choices...Choices A, C, and D do not seem to result in any medicines being prescribed so these can be eliminated. We come down to choices B and E. B tells us that a person might come to the doctor just asking for a particular medication, however does the doctor actually prescribe it? Maybe not....the doctor still has the final say. Choice E is more what we are looking for...the doctor "Gives in to a patient's demand" meaning that the doctor's own judgment is put aside here and the fact that the patient is demanding the product seen on the advertisement possibly results in an inappropriate prescription.

Answered one way, choice E strengthens the argument. If the physician does not give into the demand then no more inappropriate prescriptions. Answered the other way then it weakens, the physician gives into the demand of the patient and we have more bad prescriptions. This is the essence of "most useful to know."
Veritas Prep | GMAT Instructor

Veritas Prep Reviews
Save $100 off any live Veritas Prep GMAT Course